1
$\begingroup$

Let $M$ be a $C^3-$compact manifold and $v \in W^{2,1}_p(M\times [0,T])$ ($1\le p<\infty$) be the solution of:

$\begin{cases} \partial_t v-\Delta_{M} v=f(v), \quad M\times [0,T]\\ v(x,0)=v_0, \quad M \end{cases}$

for some $f(v)\in L^{\infty}(M\times [0,T])$ and initial data $v_0$ such that ${\vert \vert v_0 \vert \vert}_{L^\infty(M)} \le C$

I would like to prove that ${\vert \vert v(\cdot,t)-v_0(\cdot) \vert \vert}_{L^{\infty}(M)} \le Ct (*)$

However applying the Hölder inequality, and using the regularity of the time derivative of $v$, I obtain: ${\vert \vert v(\cdot,t)-v_0(\cdot) \vert \vert}_{L^p(M)} \le \hat C t^{1/q}$ without using at all the $L^{\infty}$ assumption on the initial data.

EDIT: More precisely I find applying the Hölder inequality, ${\vert \vert v(\cdot,t)-v_0(\cdot) \vert \vert}_{L^p(M)} \le \int_0^t{\vert \vert \partial_sv(\cdot,s)\vert\vert}_{L^p}\;ds \le {\vert \vert \partial_sv(\cdot,s)\vert\vert}_{L^p(L^p)} t^{1-1/p}$ I noticed that letting $p\to \infty$ then it seems reasonable to expect $(*)$. However this could be a proof if I could show that ${\vert \vert \partial_sv(\cdot,s)\vert\vert}_{L^p(L^p)}$ is bounded also for $p=\infty$ Yet I don't know how to prove this...

I think I will have to use a maximum principle technique but at this point I don't see from where to start.

I would appreciate any help or hints so I can fill in the details on my own.

DISCLAIMER: I already asked this in MSE but since I got no answer, I thought it 'd be better to post it also here.

Thanks a lot in advance!

$\endgroup$
0

2 Answers 2

3
$\begingroup$

Maybe I'm missing something, but it doesn't seem that your desired inequality can be true (even with the right-hand side of $(*)$ replaced by $C t^{\epsilon}$). It would imply uniform convergence of $v(\cdot,t)$ to $v_0(\cdot)$ as $t \rightarrow 0$, which wouldn't be possible if $v$ is continuous (say $f=0$) and $v_0$ is taken to be discontinuous.

Another way of looking at this is to test on eigenfunctions: If $v_0$ is an eigenfunction of the Laplacian with eigenvalue $-\lambda^2$ and $f = 0$, then $v(\cdot,t) - v_0(\cdot) = (e^{-\lambda^2 t} - 1) v_0(\cdot)$, but the coefficient $e^{-\lambda^2 t} - 1$ can be made bigger than $1/2$ for any positive time by simply taking $\lambda$ large enough.

$\endgroup$
2
$\begingroup$

I agree with the previous answer. Maybe the following observation for the global problem on $\mathbb R^n$ can help. Using the fundamental solution $H(t)(4π t)^{-n/2} e^{-{\vert x\vert^2/4t}}$ of the heat equation, one gets for $f=0$, \begin{multline} v(t,x)-v_0(x)=H(t) (4π t)^{-n/2}\int e^{-\vert y\vert^2/4t} \bigl(v_0(x+y)-v_0(x)\bigr) dy \\=H(t)\int e^{-π \vert z\vert^2}\bigl(v_0(x+2z\sqrt{π t})-v_0(x)\bigr) dz, \end{multline} and thus $$ v(t,x)-v_0(x)= H(t)(4π t)\int e^{-π \vert z\vert^2}\int_0^1(1-\theta)(\nabla^2 v_0)(x+\theta 2z\sqrt{π t}) z^2 dz d\theta, $$ so that, thanks to Jensen's inequality and to the translation invariance of the $L^p$-norm in $\mathbb R^n$ $$ \Vert v(t)-v_0\Vert_{L^p}\le H(t) 4π t\int e^{-π \vert z\vert^2}\vert z\vert^2\Vert\nabla^2 v_0\Vert_{L^p} dz/2=c_n t\Vert \nabla^2v_0\Vert_{L^p}, $$ where the estimates holds true for $1\le p\le +\infty$.

$\endgroup$
4
  • $\begingroup$ Maybe I should had written this in advance but $f$ depends on $v$ somehow, yet is still uniformly bounded in $L^{\infty}(M\times [0,T])$. This is why I didn't think of the fundamental solution. However, what is wrong in my approach? $\endgroup$ Jul 11, 2019 at 15:36
  • $\begingroup$ @kaithkolesidou If you want in fact boundlessness of $u_t$ the initial function $v_0$ should be from $C^{1,1}$ (first order derivatives are Lipschitz) as illustrated by this post. $\endgroup$
    – Andrew
    Jul 11, 2019 at 16:36
  • $\begingroup$ @Andrew Will I need this kind of regularity for my initial data since I want $\partial_t u$ to be Lipschitz...? Is there any theorem I miss in this case? $\endgroup$ Jul 12, 2019 at 11:56
  • 1
    $\begingroup$ @kaithkolesidou Consider the Cauchy problem for the heat equation $u_t-\Delta u=0$ in $\mathbb R^n$. The bounded solution will be smooth for $t>0$. So uniforme Lipschitz $u_t$ wrt $t$ means boundedness of $u_{tt}$. From the equation $u_{tt}=\Delta^2u$. At the initial moment $t=0$ it turns into $u_{tt}=\Delta^2v_0$. So for $u_{tt}$ to be bounded in all half space $t>0$ it is necessary that $\Delta^2v_0\in L_\infty(\mathbb R^n)$. $\endgroup$
    – Andrew
    Jul 12, 2019 at 14:14

Your Answer

By clicking “Post Your Answer”, you agree to our terms of service and acknowledge you have read our privacy policy.

Not the answer you're looking for? Browse other questions tagged or ask your own question.